Bibm@th

Forum de mathématiques - Bibm@th.net

Bienvenue dans les forums du site BibM@th, des forums où on dit Bonjour (Bonsoir), Merci, S'il vous plaît...

Vous n'êtes pas identifié(e).

Répondre

Veuillez composer votre message et l'envoyer
Nom (obligatoire)

E-mail (obligatoire)

Message (obligatoire)

Programme anti-spam : Afin de lutter contre le spam, nous vous demandons de bien vouloir répondre à la question suivante. Après inscription sur le site, vous n'aurez plus à répondre à ces questions.

Quel est le résultat de l'opération suivante (donner le résultat en chiffres)?
soixante plus vingt
Système anti-bot

Faites glisser le curseur de gauche à droite pour activer le bouton de confirmation.

Attention : Vous devez activer Javascript dans votre navigateur pour utiliser le système anti-bot.

Retour

Résumé de la discussion (messages les plus récents en premier)

bib
15-10-2017 22:46:33

Merci beaucoup pour toute votre aide.

Fred
15-10-2017 22:06:57

C'est l'intersection avec $K$, pas la réunion.

La réponse est dans ta remarque suivante, sauf que tu as faux. Une boule fermée n'est pas forcément compacte (par exemple, dans un espace vectoriel normé de dimension infinie).

F.

bib
15-10-2017 21:53:57

S'il vous plaît donc on pose $K_j= \cup_{i, x_i \in \Omega_j} \overline{B}(x_i,\delta_{x_i}) \cup K$. Pourquoi ça ne suffit pas de considérer
$K_j= \cup_{i, x_i \in \Omega_j} \overline{B}(x_i,\delta_{x_i})$? C'est pourtant un compact

Fred
15-10-2017 21:06:36

J'ai besoin de l'adhérence, parce qu'à la fin, je veux obtenir un compact. Et autant je sais que $K\cap \bar B(x,\delta)$ est compact, autant ça a toutes les chances d'être faux si je ne prends pas l'adhérence.

Tu as raison, ma méthode ne fonctionne que pour des espaces métriques. En même temps, quand on parle de $\mathcal D(\Omega)$, on a bien affaire à des ouverts de $\mathbb R^n$, non???

bib
15-10-2017 09:06:23

Fred: mais dans votre méthode vous avez utilisez les boules donc vous avez supposé qu'on est dans un espace métrique, mais il n y a pas cette hypothèse dans le texte du théorème. On utilise quand même les boules?
S'il vous plaît,
1.est-ce que je peux avoir plus de détails sur la façon de prouver le résultat par récurrence?
2. Est-ce qu'on peut choisir $\delta_x$ tel que $B(x,\delta_x) \subset \Omega_j$? Si c'est non alors pourquoi il faut $\overline{B}(x,\delta_x) \subset \Omega_j$? pourquoi l'adhérence?

leon1789
15-10-2017 08:28:45

@Fred,
je n'ai pas d'argument mathématique, mais c'est juste que je pense préalablement ainsi, par habitude : "si on veut le faire pour N ouverts, alors il faut savoir le faire pour 2 ouverts. Avec 2, cela ne peut pas être plus compliqué qu'avec N : cela évite au moins les indices ;) . Et ici, le faire pour 2 est suffisant (via une petite récurrence)."

Fred
15-10-2017 06:09:40

Pour Bib : je ne suis pas sûr que ta méthode aboutisse. Leon et moi t'en proposons deux qui fonctionnent.

Pour Leon  : le cas N=2 ne me semble pas forcément plus simple que le cas N quelconque. As-tu un argument particulier ?

bib
14-10-2017 23:22:24

Soit $\Omega_j$ un ouvert. Alors il existe une suite exhaustive de compacts $(M_l)_{l \in \mathbb{N}}$ qui couvre $\Omega_j$, c'est à dire que $\forall l \in \mathbb{N}: M_l \subset M_{l+1,j}$ et $\Omega_j= \cup_{l \in \mathbb{N}} M_{l,j}$.
On a par hypothèse que $K \subset \cup_{j=1}^n \Omega_j$, alors on a $K \subset \cup_{j=1}^n \cup_{l \in \mathbb{N}} M_{l,j}$.
Ma difficulté est de déduire la forme du compact $K_j$ dont on a besoin. Est-ce qu'il y a une issue? S'il vous plaît.

leon1789
14-10-2017 21:35:21

Salut,
on peut faire une preuve par récurrence sur N : le coeur du problème réside dans le cas N=2, car l'hérédité ne pose pas de souci particulier (en groupant les deux derniers ouverts d'indice N-1 et N, on applique l'hypothèse de récurrence au rang N-1 et au rang 2).

Fred
14-10-2017 21:30:35

Bonsoir,

  Voici une preuve, pas forcément la meilleure possible!
Soit $x\in K$.
On note $\delta_x>0$ tel que, pour tout $j=1,\dots, N$, si $x\in \Omega_j$, alors $\bar B(x,\delta_x)\subset\Omega_j$ (c'est possible de trouver un tel $\delta_x$ car les $\Omega_j$ sont ouverts).
On a $K\subset\bigcup_{x\in K}B(x,\delta_x)$. Puisque $K$ est compact, on peut extraire de ce recouvrement ouvert un recouvrement fini
$K\subset B(x_1,\delta_{x_1})\cup\dots B(x_p,\delta_{x_p})$.

On note alors $K_j=\bigcup_{i;\ x_i\in \Omega_j}\bar B(x_i,\delta_{x_i})\cap K$. Chaque $K_j$ est compact, car réunion finie de compacts. De plus, $K\subset\bigcup_{j=1}^N K_j$ (on a même égalité), car si $x\in K$, alors $x\in B(x_i,\delta_{x_i})$ pour un certain $i$ et donc $x\in K_j$ où $j$ est tel que $x_i\in\Omega_j$.
Enfin, $K_j\subset\Omega_j$ car par définition de $\delta_{x_i}$, si $x_i\in \Omega_j$, alors $\bar B(x_i,\delta_{x_i})\subset \Omega_j$.

F.

bib
14-10-2017 11:05:42

Bonjour,
je cherche une démonstration pour le théorème suivant: soit $K$ un compact tel que $K \subset \cup_{j=1}^N \Omega_j$ tels que $(\Omega_j)$ sont des ouverts. Alors, il existe des compacts $K_j \subset \Omega_j$ tels que $K \subset \cup_{j=1}^N K_j$.

Ce théorème est venu après la définition suivante: soit $\Omega$ un ouvert et soit $(\Omega_j)$ une suite exhaustive d'ouverts qui couvre $\Omega$. On dit d'une suite $(\varphi_j)$ de $\mathcal{D}(\Omega)$ qu'elle est "truncation sequence" si elle vérifie: $\forall j, 0 \leq \varphi_j \leq 1$ et $\varphi_j=1$ au voisinage de $\overline{\Omega}_j$.
Voici l'idée que j'ai travaillé. On suppose que $K \subset \cup_{j=1}^N \Omega_j$ où $\Omega_j$ est un ouvert pour tout $j$, et on sait que tout ouvert admet une suite exhaustive de compacts, c'est à dire que pour tout $j$, il existe une famille exhaustive de compacts $M_{i,j}$ tels que
$\Omega_j=\cup_{i=1}^N M_{i,j}$. Par conséquent, on a $K \subset \cup_{i,j=1}^N M_{i,j}$.
Merci par avance pour votre aide.

Pied de page des forums